Cranial Trauma Flashcards

1
Q

Which one of the following approximate ratios for the proportion of traumatic brain injuries that are mild, moderate or severe is most accurate?
a. 22:1.5:1
b. 20:3.5:1
c. 20:2.5:2
d. 18:3.5:3
e. 18:4.5:2

A

a. 22:1.5:1

a—22 mild TBI: 1.5 moderate TBI: 1
severe TBI
A World Health Organization (WHO) systematic review of the mTBI literature found that
70-90% of TBI was mild in nature and that
hospital-treated mild TBI was approximately
100-300 per 100,000 in the studies it reviewed.
However, given the undertreatment and reporting of mild TBI, the WHO estimated that the
true yearly incidence was likely 600 per
100,000. Average estimated incidence of TBI in
the United States 577 per 100,000, of which
465 per 100,000 were treated in the emergency
department and released, 94 per 100,000 were
hospitalized and discharged alive, and 18 per
100,000 died. In the UK, every year 1500 per
100,000 of the population attend emergency
departments with a head injury, 225-300 per
100,000 are admitted to hospital, 10-15 per
100,000 are admitted to neurosurgical units and
6-10 per 100,000 die from TBI. The aforementioned studies constitute some of the best epidemiological data on incidence of TBI, but the data
likely grossly underestimates the incidence of
mTBI. European epidemiological studies that
calculated a TBI severity ratio of 22 mild TBI:
1.5 moderate TBI: 1 severe TBI (i.e. 90% of
TBI is mild). In general, TBI is much more frequent in males than females (1.4:1), highest
among young children aged 0-4 (1337 per
100,000) and older adolescence aged 15-19 (896
per 100,000). Older adults aged 75 and above also
have a high rate of TBI (932 per 100,000) and
they account for the highest rate of TBIassociated hospitalizations (339 per 100,000)
and death (57 per 100,000). This pattern of high
rates of TBI in early childhood, late adolescence,
and in the elderly has been shown in many
population-based studies. The relative risk of a
second TBI among those with an earlier TBI
was 2.8-3 times greater than the non-injured sample. Additionally, in those that sustained a
second head injury the risk of sustaining a third
head injury was 7.8-9.3 times that of an initial
head injury in the population. Alcohol is involved
in one third to two thirds of cases, and 20% of
those with TBI following motor vehicle
collisions

atic review of the mTBI literature found that 70-90% of TBI was mild in nature and that hospital-treated mild TBI was approximately 100-300 per 100,000 in the studies it reviewed. However, given the undertreatment and report- ing of mild TBI, the WHO estimated that the true yearly incidence was likely 600 per 100,000. Average estimated incidence of TBI in the United States 577 per 100,000, of which 465 per 100,000 were treated in the emergency department and released, 94 per 100,000 were hospitalized and discharged alive, and 18 per 100,000 died. In the UK, every year 1500 per 100,000 of the population attend emergency departments with a head injury, 225-300 per 100,000 are admitted to hospital, 10-15 per 100,000 are admitted to neurosurgical units and 6-10 per 100,000 die from TBI. The aforemen- tioned studies constitute some of the best epide- miological data on incidence of TBI, but the data likely grossly underestimates the incidence of mTBI. European epidemiological studies that calculated a TBI severity ratio of 22 mild TBI: 1.5 moderate TBI: 1 severe TBI (i.e. 90% of TBI is mild). In general, TBI is much more fre- quent in males than females (1.4:1), highest among young children aged 0-4 (1337 per 100,000) and older adolescence aged 15-19 (896 per 100,000). Older adults aged 75 and above also have a high rate of TBI (932 per 100,000) and they account for the highest rate of TBI- associated hospitalizations (339 per 100,000) and death (57 per 100,000). This pattern of high rates of TBI in early childhood, late adolescence, and in the elderly has been shown in many population-based studies. The relative risk of a second TBI among those with an earlier TBI was 2.8-3 times greater than the non-injured sample. Additionally, in those that sustained a second head injury the risk of sustaining a third head injury was 7.8-9.3 times that of an initial head injury in the population. Alcohol is involved in one third to two thirds of cases, and 20% of those with TBI following motor vehicle collisions

How well did you know this?
1
Not at all
2
3
4
5
Perfectly
2
Q

Which one of the following statements regarding the pathophysiology of traumatic brain injury is LEAST accurate?
a. An acute extradural hematoma is a type of primary brain injury
b. Diffuse axonal injury is a type of second- ary brain injury
c. Cerebral contusions are a type of primary brain injury
d. Glutamate excitotoxicity is a type of secondary brain injury
e. Hypoxia is a cause of secondary brain injury

A

b. Diffuse axonal injury is a type of second- ary brain injury

TBI is commonly subdivided into primary and secondary injury. Primary injury results from the
mechanical forces on the brain from a combination
of direct impact, penetrating injuries and shock
waves, and acceleration/deceleration phenomena.
Examples of primary injury are diffuse axonal
injury, cerebral contusions/hemorrhage, and
extra-axial hemorrhage (EDH, SDH, SAH).
Secondary brain injury is non-mechanical in
nature and due to local (e.g. cerebral edema,
regional loss of cerebral autoregulation, anaerobic
glycolysis/lactic acidosis, glutamate excitotoxicity)
and systemic (e.g. hypoxia, hypotension, hypoglycemia) consequences of the primary injury which
aggravate it further. An intermediate set of processes, such as apoptosis and axonal retraction,
occur in a delayed fashion (overlapping with time
course of secondary injury) but arise from the primary injury. In general, primary brain injury is not
treatable (except for hematoma evacuation) and
its magnitude is a limiting factor distinguishing
survivable from non-survivable brain injury. The
major focus of TBI management is thus on limiting/preventing secondary brain injury as
much as possible to reduce unfavorable outcomes;
this is achieved by ensuring adequate oxygenation/
gas transfer, organ perfusion and control of ICP.
More recently, tertiary injuries (iatrogenic) have
been recognized to contribute to outcome just as
strongly as primary or secondary injury. Examples
of tertiary injury include complications from
a prolonged intensive care unit (ICU) stay
(e.g. ventilator-associated pneumonia, line sepsis,
decubitus ulcers, or medication administration
errors), as well as complications of brain treatment
per se, such as transfusion-related acute lung
injury, post-operative neurosurgical infections,
vasopressor-related ischemia, or hyperosmolar
renal failure

TBI is commonly subdivided into primary and sec- ondary injury. Primary injury results from the mechanical forces on the brain from a combination of direct impact, penetrating injuries and shock waves, and acceleration/deceleration phenomena. Examples of primary injury are diffuse axonal injury, cerebral contusions/hemorrhage, and extra-axial hemorrhage (EDH, SDH, SAH). Secondary brain injury is non-mechanical in nature and due to local (e.g. cerebral edema, regional loss of cerebral autoregulation, anaerobic glycolysis/lactic acidosis, glutamate excitotoxicity) and systemic (e.g. hypoxia, hypotension, hypogly- cemia) consequences of the primary injury which aggravate it further. An intermediate set of pro- cesses, such as apoptosis and axonal retraction, occur in a delayed fashion (overlapping with time course of secondary injury) but arise from the pri- mary injury. In general, primary brain injury is not treatable (except for hematoma evacuation) and its magnitude is a limiting factor distinguishing survivable from non-survivable brain injury. The major focus of TBI management is thus on

limiting/preventing secondary brain injury as much as possible to reduce unfavorable outcomes; this is achieved by ensuring adequate oxygenation/ gas transfer, organ perfusion and control of ICP. More recently, tertiary injuries (iatrogenic) have been recognized to contribute to outcome just as strongly as primary or secondary injury. Examples of tertiary injury include complications from a prolonged intensive care unit (ICU) stay (e.g. ventilator-associated pneumonia, line sepsis, decubitus ulcers, or medication administration errors), as well as complications of brain treatment per se, such as transfusion-related acute lung injury, post-operative neurosurgical infections, vasopressor-related ischemia, or hyperosmolar renal failure.

How well did you know this?
1
Not at all
2
3
4
5
Perfectly
3
Q

Which one of the following statements regar- ding prognosis in TBI is LEAST accurate?
a. Mortality rate in severe TBI (GCS 3-8) is approximately 40%
b. Hypoxia and hypoglycemia are the extra- cranial insults which most strongly affect prognosis after TBI
c. Prognostication based on CT head appearance may be done using Marshall or Rotterdam classifications
d. Mortality rate in those with mild TBI (GCS 13-15) is <1% overall
e. Mortality rate at 14 days in those with GCS 13 and bilaterally reactive pupils after a significant, isolated head injury maybeupto30%

A

b. Hypoxia and hypoglycemia are the extra- cranial insults which most strongly affect

prognosis after TBI

Functional recovery from TBI is classified on the
Glasgow Outcome Scale and is commonly
dichotomized into favorable outcome (5¼good
recovery; 4¼moderate disability/independent)
versus unfavorable outcome (3¼severe disability/dependent; 4¼vegetative state; 1¼death).
The overwhelming majority of patients with mild
TBI (GCS 13-15) make a good recovery even
though cognitive deficits and symptoms are common in the acute stage, and overall mortality is
<1% (reflects the fact that the vast majority of
mild TBIs are GCS 15; mortality rate may be
up to 30% in those with GCS 13 and reactive
pupils but significant brain injury/hematoma on
CT head). Mortality rate in moderate TBI is
approximately 15% overall. Patients presenting
in coma with severe TBI have a 40% mortality
rate and a further 20% survive with major disability. Predicting outcome for an individual patient
is, however, notoriously difficult and a clear prognosis often emerges only over the days and weeks following injury. MRI of the brain may assist with
prognostication of outcome. However, early
prognostication and utility in deciding aggressiveness of treatment is plagued by several factors:
Inaccurate GCS recording, alcohol/drug intoxication, sedative drugs, intubation (no verbal
GCS score), associated trauma preventing assessment of GCS (e.g. maxillo-facial injury), and
uncorrected systemic insults (hypotension, hypoxia, hypothermia) which affect consciousness.
The IMPACT meta-analysis reviewed reversible
insults present on admission and their potential to
influence outcome by exacerbating secondary
injury. Hypoxia, hypotension (SBP <90 mmHg)
and hypothermia (<35 °C) were strongly associated with poor outcomes, with hypoxia and hypotension having synergistic effects. Prognostic
models (e.g. IMPACT or CRASH) derived from
large prospective datasets have shown that age,
severity of primary injury (measured by GCS,
pupillary reaction, and CT scan appearances such
as traumatic SAH/IVH, cistern effacement, epidural masses, midline shift) and major secondary
insults including hypotension, hypoxia and hypothermia are the principal risk factors for death and
long-term neurological morbidity. CT appearances alone can also be used for prognostication
via Marshall or Rotterdam CT criteria. For illustrative purposes only, CRASH prognostic calculators is shown for isolated head injury (bilateral
reactive pupils, CT evidence of petechial hemorrhage, SAH, cistern effacement and MLS without
hematoma) scenarios below as ranges of outcome
for mild (GCS 13-14), moderate (9-12) and
severe (3-8) TBI groups. The CRASH score also
incorporates adjustment according to high
income (shown) versus low-middle income countries, and GCS 14 or less. IMPACT only considered moderate and severe head injuries (but only
motor score is put in) and can be used with or
without CT (Marshall) criteria

Functional recovery from TBI is classified on the Glasgow Outcome Scale and is commonly dichotomized into favorable outcome (51⁄4good recovery; 4 1⁄4 moderate disability/independent) versus unfavorable outcome (31⁄4severe disabil- ity/dependent; 4 1⁄4 vegetative state; 1 1⁄4 death). The overwhelming majority of patients with mild TBI (GCS 13-15) make a good recovery even though cognitive deficits and symptoms are com- mon in the acute stage, and overall mortality is <1% (reflects the fact that the vast majority of mild TBIs are GCS 15; mortality rate may be up to 30% in those with GCS 13 and reactive pupils but significant brain injury/hematoma on CT head). Mortality rate in moderate TBI is approximately 15% overall. Patients presenting in coma with severe TBI have a 40% mortality rate and a further 20% survive with major disabil- ity. Predicting outcome for an individual patient is, however, notoriously difficult and a clear prog- nosis often emerges only over the days and weeks
following injury. MRI of the brain may assist with prognostication of outcome. However, early prognostication and utility in deciding aggres- siveness of treatment is plagued by several factors: Inaccurate GCS recording, alcohol/drug intoxi- cation, sedative drugs, intubation (no verbal GCS score), associated trauma preventing assess- ment of GCS (e.g. maxillo-facial injury), and uncorrected systemic insults (hypotension, hyp- oxia, hypothermia) which affect consciousness. The IMPACT meta-analysis reviewed reversible insults present on admission and their potential to influence outcome by exacerbating secondary injury. Hypoxia, hypotension (SBP <90 mmHg) and hypothermia (<35 °C) were strongly associ- ated with poor outcomes, with hypoxia and hypo- tension having synergistic effects. Prognostic models (e.g. IMPACT or CRASH) derived from large prospective datasets have shown that age, severity of primary injury (measured by GCS, pupillary reaction, and CT scan appearances such as traumatic SAH/IVH, cistern effacement, epi- dural masses, midline shift) and major secondary insults including hypotension, hypoxia and hypo- thermia are the principal risk factors for death and long-term neurological morbidity. CT appear- ances alone can also be used for prognostication via Marshall or Rotterdam CT criteria. For illus- trative purposes only, CRASH prognostic calcu- lators is shown for isolated head injury (bilateral reactive pupils, CT evidence of petechial hemor- rhage, SAH, cistern effacement and MLS without hematoma) scenarios below as ranges of outcome for mild (GCS 13-14), moderate (9-12) and severe (3-8) TBI groups. The CRASH score also incorporates adjustment according to high income (shown) versus low-middle income coun- tries, and GCS 14 or less. IMPACT only consid- ered moderate and severe head injuries (but only motor score is put in) and can be used with or without CT (Marshall) criteria.

How well did you know this?
1
Not at all
2
3
4
5
Perfectly
4
Q

Which one of the following statements regarding mild traumatic brain injury (GCS 13-15) patients presenting to emergency departments are LEAST accurate?
a. Loss of consciousness is not a requirement for diagnosis
b. It is associated with increased glutamate release, and a hyperglycotic and hyperme- tabolic state in brain tissue
c. Clinically significant brain injury is pre- sent in 10-15% of cases
d. The majority of patients are symptom free by 7-10 days post-concussion
e. Approximately 0.5-1% result in death or require neurosurgical intervention due to underlying significant brain injury

A

c. Clinically significant brain injury is pre- sent in 10-15% of cases

Concussion (mild TBI) is a traumatically induced
alteration in consciousness (confusion, amnesia
with or without an associated loss of consciousness) due to a non-penetrating injury. It usually
occurs immediately following the blow or within
minutes of it. The majority do not have loss of consciousness hence fail to be recognized. Levels of
glutamate rise after concussion and the brain
enters a hyperglycotic and hypermetabolic state
which may persist for 7-10 days after injury (i.e.
the period after which the vast majority of patients
are symptom free again), and may make the brain
more susceptible to a second impact as altered
cerebral autoregulation may produce much more
severe sequelae (malignant cerebral edema resistant to treatment and almost certainly fatal). Mild
TBI has been classified as GCS of 13-15 (or more
recently 14-15 depending on series) but the vast
majority are initially GCS 15/15 hence other
features must be used to assess risk of further deterioration due to underlying significant brain injury
requiring treatment or observation (i.e. a proportion may actually be occult moderate/potentially
severe TBI: “talk and die patients” with a lucid
interval before deterioration). Clinical grading
systems for concussion based on duration/
presence of confusion, post-traumatic amnesia
and LOC have been used, but key concerns generally focus on the need for and timing of neuroimaging for which more useful rules incorporating these features exist (e.g. Canadian CT head
rules, New Orleans Criteria). In a recent systematic
review of 23,079 adults presenting with minor head
trauma (GCS 13-15 who appear well on examination). The prevalence of severe intracranial injury
(subdural, epidural, ventricular or parenchymal
hematoma, subarachnoid hemorrhage, herniation,
or depressed skull fracture, small intracranial hemorrhages requiring observation in the hospital,
neurosurgical evaluation, or operative intervention) was 7.1% (95% CI, 6.8-7.4%) and the prevalence of injuries leading to death or requiring
neurosurgical intervention was 0.9% (95% CI,
0.78-1.0%). In those with abnormal CT head not
requiring surgery or those with normal CT head
but if GCS <15, seizures or coagulopathy inpatient
observation for 24 h is recommended due to risk of
developing intracranial complications (e.g. cerebral
swelling, delayed hematoma) and need for repeat
CT head before discharge. In those with normal
CT head and normal GCS with none or mild
symptoms may be able to be observed at home
by a responsible adult aware of signs requiring
immediate medical assessment. Further management on discharge relates to post-concussion
syndrome, risk of second impact syndrome (particularly return to play guidelines in athletes and
contraindications to returning to contact sport),
risk of post-traumatic epilepsy and, in those with
multiple concussions, risk of chronic traumatic
encephalopathy

Concussion (mild TBI) is a traumatically induced alteration in consciousness (confusion, amnesia with or without an associated loss of conscious- ness) due to a non-penetrating injury. It usually occurs immediately following the blow or within minutes of it. The majority do not have loss of con- sciousness hence fail to be recognized. Levels of glutamate rise after concussion and the brain enters a hyperglycotic and hypermetabolic state which may persist for 7-10 days after injury (i.e. the period after which the vast majority of patients are symptom free again), and may make the brain more susceptible to a second impact as altered cerebral autoregulation may produce much more severe sequelae (malignant cerebral edema resis- tant to treatment and almost certainly fatal). Mild TBI has been classified as GCS of 13-15 (or more recently 14-15 depending on series) but the vast majority are initially GCS 15/15 hence other features must be used to assess risk of further dete- rioration due to underlying significant brain injury requiring treatment or observation (i.e. a propor- tion may actually be occult moderate/potentially severe TBI: “talk and die patients” with a lucid interval before deterioration). Clinical grading systems for concussion based on duration/ presence of confusion, post-traumatic amnesia and LOC have been used, but key concerns
generally focus on the need for and timing of neu- roimaging for which more useful rules incorporat- ing these features exist (e.g. Canadian CT head rules, New Orleans Criteria). In a recent systematic review of 23,079 adults presenting with minor head trauma (GCS 13-15 who appear well on examina- tion). The prevalence of severe intracranial injury (subdural, epidural, ventricular or parenchymal hematoma, subarachnoid hemorrhage, herniation, or depressed skull fracture, small intracranial hem- orrhages requiring observation in the hospital, neurosurgical evaluation, or operative interven- tion) was 7.1% (95% CI, 6.8-7.4%) and the prev- alence of injuries leading to death or requiring neurosurgical intervention was 0.9% (95% CI, 0.78-1.0%). In those with abnormal CT head not requiring surgery or those with normal CT head but if GCS <15, seizures or coagulopathy inpatient observation for 24 h is recommended due to risk of developing intracranial complications (e.g. cerebral swelling, delayed hematoma) and need for repeat CT head before discharge. In those with normal CT head and normal GCS with none or mild symptoms may be able to be observed at home by a responsible adult aware of signs requiring immediate medical assessment. Further manage- ment on discharge relates to post-concussion syndrome, risk of second impact syndrome (partic- ularly return to play guidelines in athletes and contraindications to returning to contact sport), risk of post-traumatic epilepsy and, in those with multiple concussions, risk of chronic traumatic encephalopathy.

How well did you know this?
1
Not at all
2
3
4
5
Perfectly
5
Q

A football player sustains a head injury during a game. On examination he is GCS 15/15. Which one of the following statements about complications following concussion is most accurate?
a. Second impact syndrome is a common cause of rapid fatal brain swelling after concussion b. The commonest symptoms of post- concussion syndrome are ongoing cognitive impairment
c. Chronic traumatic encephalopathy is characterized clinically by neurodegeneration
d. The risk of late seizures (post-traumatic epilepsy) in those with mild traumatic brain injury is thought to be threefold higher in the next 5 years.
e. Dysfunction of cerebral autoregulation after an initial concussion is thought to underlie the risk for second impact syndrome

A

e. Dysfunction of cerebral autoregulation after an initial concussion is thought to underlie the risk for second impact syndrome

How well did you know this?
1
Not at all
2
3
4
5
Perfectly
6
Q

Which one of the following statements regard- ing return to play of an athlete who has sus- tained a concussion is LEAST accurate?
a. An athlete cannot return to play that day even once concussion symptoms have cleared
b. Risk of a second concussion is particularly increased in the 10 days following the first concussion
c. Return to play guidelines are based on risk of second impact syndrome
d. Have full resolution of their symptoms (off medication) and approval by an LHCP to return to play
e. Neuroimaging should be obtained based on the presence of risk factors for clini- cally significant brain injuries

A

c. Return to play guidelines are based on risk of second impact syndrome

AAN sports concussion guidelines 2013 advised
that any athlete suspected of having sustained a
concussion should be immediately removed from
play to minimize the risk for further injury. The
risk of further injury refers to the evidence that a
single concussion predisposes to a second one,
and this risk of a second concussion is particularly increased in the 10 days following the first
concussion. The reason for this increased risk for
a second injury is unknown, but given that it
mirrors the time taken for >90% of those with
concussion to become symptom-free again (i.e.
7-10 days) the most likely hypothesis is that
impaired cognition or physical reflexes due to
the first concussion increase the player’s susceptibility to injury. Due to a lack of robust evidence
regarding their respective mechanisms concerns
regarding the risks of repetitive head injury (second impact syndrome or chronic traumatic
encephalopathy) are not the basis for current
recommendations, although the general principle of avoiding a second impact while still
symptomatic from the first may be necessary
but not sufficient to avoid second impact syndrome based on current concepts regarding
its pathophysiology. More specific guidance is
outlined below:
* Players who experience symptoms suggestive
of concussion, such as blurry or double vision,
confusion, dizziness, headache, nausea, memory
loss, or other cognitive or behavioral problems,
must have full resolution of their symptoms
(off medication) and approval by an LHCP to
return to play. Supplemental neurocognitive
testing, including comparisons with agematched normal profiles or a patient’s baseline
profile can be used to aid decision making
* An athlete cannot return to play that day if a
concussion had been diagnosed, even if symptoms had cleared. This may be inadvertently
circumvented if the athlete hides their symptoms and has a normal examination, or player
with a witnessed head injury whose concussive
symptoms don’t appear until after the game
(who would have already been exposed to the
risk of a second impact)
* Concussion is a clinical diagnosis. In a proportion of those with concussion, a CT head
should be obtained based on the presence of
risk factors (e.g. Canadian CT head rules,
New Orleans criteria) to rule out clinically important brain injury requiring admission to
hospital for observation or neurosurgery
* Athletes with multiple concussions and continued impairment, should undergo formal neurologic and cognitive assessment and be
counseled on the risk for developing chronic
neurobehavioral or cognitive impairment and
retirement recommended (lower threshold
for professional vs. amateur athletes)

that any athlete suspected of having sustained a concussion should be immediately removed from play to minimize the risk for further injury. The risk of further injury refers to the evidence that a single concussion predisposes to a second one, and this risk of a second concussion is particu- larly increased in the 10 days following the first concussion. The reason for this increased risk for a second injury is unknown, but given that it mirrors the time taken for >90% of those with concussion to become symptom-free again (i.e. 7-10days) the most likely hypothesis is that impaired cognition or physical reflexes due to the first concussion increase the player’s suscep- tibility to injury. Due to a lack of robust evidence regarding their respective mechanisms concerns regarding the risks of repetitive head injury (sec- ond impact syndrome or chronic traumatic encephalopathy) are not the basis for current recommendations, although the general prin- ciple of avoiding a second impact while still symptomatic from the first may be necessary but not sufficient to avoid second impact
syndrome based on current concepts regarding its pathophysiology. More specific guidance is outlined below:
* Players who experience symptoms suggestive
of concussion, such as blurry or double vision, confusion, dizziness, headache, nausea, memory loss, or other cognitive or behavioral problems, must have full resolution of their symptoms (off medication) and approval by an LHCP to return to play. Supplemental neurocognitive testing, including comparisons with age- matched normal profiles or a patient’s baseline profile can be used to aid decision making
* An athlete cannot return to play that day if a concussion had been diagnosed, even if symp- toms had cleared. This may be inadvertently circumvented if the athlete hides their symp- toms and has a normal examination, or player with a witnessed head injury whose concussive symptoms don’t appear until after the game (who would have already been exposed to the risk of a second impact)
* Concussion is a clinical diagnosis. In a propor- tion of those with concussion, a CT head should be obtained based on the presence of risk factors (e.g. Canadian CT head rules, New Orleans criteria) to rule out clinically important brain injury requiring admission to
hospital for observation or neurosurgery
*Athleteswithmultipleconcussionsandcontin- ued impairment, should undergo formal neu- rologic and cognitive assessment and be counseled on the risk for developing chronic neurobehavioral or cognitive impairment and retirement recommended (lower threshold
for professional vs. amateur athletes)

How well did you know this?
1
Not at all
2
3
4
5
Perfectly
7
Q

In the UK, which one of the following head injury scenarios necessitates a CT scan within 1 h of being identified (NICE head injury guideline CG176)?
a. More than 30 min of retrograde amnesia of events immediately before the head injury
b. Loss of consciousness and dangerous mechanism of head injury
c. Isolated post-traumatic seizure
d. Amnesia since the injury and history of
easily bruising (currently on aspirin)
e. Loss of consciousness since the head
injury and age >65 years

A

c—Isolated post-traumatic seizure

How well did you know this?
1
Not at all
2
3
4
5
Perfectly
8
Q

Which one of the following statements regarding CT head scanning in those with mild head injury is most accurate?
a. New Orleans criteria apply to head injured patients with GCS 13-15 and loss of consciousness with no focal deficit on neurological examination
b. Canadian CT head rule high risk group includes those with GCS 13-15 and are aged 70 or older
c. Canadian CT head rule medium risk cri- teria include dangerous mechanism of head injury
d. Canadian CT head rule medium risk cri- teria include two or more episodes of vomiting
e. New Orleans criteria apply to GCS 15 head injured patients without loss of con- sciousness and with a normal examination

A

c. Canadian CT head rule medium risk cri- teria include dangerous mechanism of head injury

Many physicians follow a set of criteria for selecting
which patients should receive a head CT following
mild TBI (concussion). The Canadian CT Head
Rule (CCR) was derived from a study of 3121
patients presenting to 10 Canadian hospitals with
a GCS score of 13-15/15 after head injury (excluding those<16 years, bleeding disorders or warfarin,
obvious open skull fracture). CT head is mandatory
if one or more of the following high-risk criteria for
neurosurgical intervention are present: (1) GCS
score less than 15 at 2 h after head injury; (2) suspected open or depressed skull fracture; (3) any sign
of basal skull fracture (e.g. hemotympanum, “raccoon” eyes, CSF otorrhea/rhinorrhea, Battle’s
sign); (4) two or more episodes of vomiting; and
(5) patientis 65 years of age or older.CT headis also
recommended in patients in the medium-risk category who may have clinically important brain injuries that may require admission: (1) greater than
30 min of retrograde amnesia or (2) injury via a
“dangerous mechanism” (e.g. motor vehicle accident versus pedestrian, ejection from motor vehicle,
fall from greater than 3 feet or down five or more
stairs). The New Orleans criteria for CT head in
mild TBI applies to emergency department patients
with aGCS 15/15 onlywithLOC and a normal neurological examination. In this situation, CT head
should be performed if any one of the following risk
factors present: age >60 years, headache, vomiting,
drug/alcohol intoxication, persistent anterograde
amnesia, post-traumatic seizure, and evidence of
trauma above the clavicles. In a recent systematic
review of 23,079 adults presenting with minor head trauma (GCS 13-15 who appear well on examination). The prevalence of severe intracranial injury
(requiring prompt intervention) was 7.1% (95%
CI, 6.8-7.4%) and the prevalence of injuries leading
to death or requiring neurosurgical intervention
was 0.9% (95% CI, 0.78-1.0%). Features most predictive of severe intracranial injury on CT were
examination findings suggestive of skull fracture,
GCS score 13/15, 2 or more vomiting episodes,
any decline in GCS and pedestrians struck by motor
vehicles. Absence of any of the features of the CanadianCTHeadRule (high andmedium risk)lowered
the probability of severe injury to 0.31% (95% CI,
0-4.7%). The absence of any New Orleans Criteria
findings lowered the probability of severe intracranial injury to 0.61% (95% CI, 0.08-6.0%)

Many physicians follow a set of criteria for selecting which patients should receive a head CT following mild TBI (concussion). The Canadian CT Head Rule (CCR) was derived from a study of 3121 patients presenting to 10 Canadian hospitals with a GCS score of 13-15/15 after head injury (exclud- ing those <16 years, bleeding disorders or warfarin, obvious open skull fracture). CT head is mandatory if one or more of the following high-risk criteria for neurosurgical intervention are present: (1) GCS score less than 15 at 2 h after head injury; (2) sus- pected open or depressed skull fracture; (3) any sign of basal skull fracture (e.g. hemotympanum,
“raccoon” eyes, CSF otorrhea/rhinorrhea, Battle’s sign); (4) two or more episodes of vomiting; and (5) patient is 65 years of age or older. CT head is also recommended in patients in the medium-risk cate- gory who may have clinically important brain inju- ries that may require admission: (1) greater than 30 min of retrograde amnesia or (2) injury via a “dangerous mechanism” (e.g. motor vehicle acci- dent versus pedestrian, ejection from motor vehicle, fall from greater than 3 feet or down five or more stairs). The New Orleans criteria for CT head in mild TBI applies to emergency department patients with a GCS 15/15 only with LOC and a normal neu- rological examination. In this situation, CT head should be performed if any one of the following risk factors present: age >60 years, headache, vomiting, drug/alcohol intoxication, persistent anterograde amnesia, post-traumatic seizure, and evidence of trauma above the clavicles. In a recent systematic review of 23,079 adults presenting with minor head trauma (GCS 13-15 who appear well on examina- tion). The prevalence of severe intracranial injury (requiring prompt intervention) was 7.1% (95% CI, 6.8-7.4%) and the prevalence of injuries leading to death or requiring neurosurgical intervention was 0.9% (95% CI, 0.78-1.0%). Features most pre- dictive of severe intracranial injury on CT were examination findings suggestive of skull fracture, GCS score 13/15, 2 or more vomiting episodes, any decline in GCS and pedestrians struck by motor vehicles. Absence of any of the features of the Cana- dian CT Head Rule (high and medium risk) lowered the probability of severe injury to 0.31% (95% CI, 0-4.7%). The absence of any New Orleans Criteria findings lowered the probability of severe intracra- nial injury to 0.61% (95% CI, 0.08-6.0%).

How well did you know this?
1
Not at all
2
3
4
5
Perfectly
9
Q

Which one of the following statements regarding moderate traumatic brain injury is LEAST accurate? Which one of the following statements regarding moderate traumatic brain injury is LEAST accurate?
a. Include head injuries with GCS 9-13
b. 30% chance of having a brain lesion
(intra- or extra-axial)
c. Mortality is around 30%
d. Account for 5-7% of head injury atten-
dances in the emergency department
e. Includes patients who “talk and die”

A

c. Mortality is around 30%

TBI with GCS scores of 9-12 is considered moderate, though some now consider those with a
GCS of 13 within this category too given that a
third have abnormal CT head findings. They
account for 5-7% of head injury attendances in
the emergency department (22 mild: 1.5 moderate: 1 severe) and affects the young adult population involved in traffic accidents, is associated
with alcohol or illicit drugs, and with extracranial
injuries. Individuals with moderate TBI have
approximately a 30% chance of having a brain
lesion (intra- or extra-axial), a 30% chance that
such injuries progress in their volume or mass
effect (new bleeding, rebleeding, edema) and a
30% chance that these individuals suffer deterioration or worsening in their neurological status.
Mortality in moderate TBI is around 15%,
>50% have cognitive sequelae and only 20%
recover without significant disability. Most “talk
and die” patients (i.e. lucid interval; patients presenting with a verbal GCS score 3 who were
thought to have sustained a survivable head injury
who later deteriorate and die to due potentially
treatable head injury) should also be in the moderate TBI category. The authors concluded that
morbidity and mortality in these patients might
be reduced by early diagnosis and more aggressive treatment of raised ICP. More recent studies
have shown that patients who “talk and die” are
most frequently adult men and the most common
mechanisms of trauma are falls, motor vehicle
accidents and violence. In these studies, the average GCS at admission to emergency department was 14 and the most frequent intracranial injuries
were acute subdural hematoma, diffuse cerebral
edema and cerebral contusion. In about 14% of
these patients with GCS 13 at admission, initial
CT was normal but became abnormal during
hospitalization, especially because of development of diffuse cerebral edema. Among the most
important factors relating to death are: delays in
diagnosis of lesion through CT scan (as initially
appear well), delays in the transfer to a specialized
center, failure to identify risk factors for deterioration, inadequate prevention of secondary
injury, inappropriate correction of underlying
coagulopathy and loss of the opportunity for
definitive neurosurgical treatment

TBI with GCS scores of 9-12 is considered mod- erate, though some now consider those with a GCS of 13 within this category too given that a third have abnormal CT head findings. They account for 5-7% of head injury attendances in the emergency department (22 mild: 1.5 moder- ate: 1 severe) and affects the young adult popula- tion involved in traffic accidents, is associated with alcohol or illicit drugs, and with extracranial injuries. Individuals with moderate TBI have approximately a 30% chance of having a brain lesion (intra- or extra-axial), a 30% chance that such injuries progress in their volume or mass effect (new bleeding, rebleeding, edema) and a 30% chance that these individuals suffer deterio- ration or worsening in their neurological status. Mortality in moderate TBI is around 15%, >50% have cognitive sequelae and only 20% recover without significant disability. Most “talk and die” patients (i.e. lucid interval; patients pre- senting with a verbal GCS score !3 who were thought to have sustained a survivable head injury who later deteriorate and die to due potentially treatable head injury) should also be in the mod- erate TBI category. The authors concluded that morbidity and mortality in these patients might be reduced by early diagnosis and more aggres- sive treatment of raised ICP. More recent studies have shown that patients who “talk and die” are most frequently adult men and the most common mechanisms of trauma are falls, motor vehicle accidents and violence. In these studies, the aver- age GCS at admission to emergency department
was 14 and the most frequent intracranial injuries were acute subdural hematoma, diffuse cerebral edema and cerebral contusion. In about 14% of these patients with GCS 13 at admission, initial CT was normal but became abnormal during hospitalization, especially because of develop- ment of diffuse cerebral edema. Among the most important factors relating to death are: delays in diagnosis of lesion through CT scan (as initially appear well), delays in the transfer to a specialized center, failure to identify risk factors for deterio- ration, inadequate prevention of secondary injury, inappropriate correction of underlying coagulopathy and loss of the opportunity for definitive neurosurgical treatment.

How well did you know this?
1
Not at all
2
3
4
5
Perfectly
10
Q

Which one of the following is a biomarker for traumatic brain injury?
a. GFAP
b. TP53
c. ATRX
d. VEGF
e. IDH-1

A

a. GFAP

In future, early biomarkers may facilitate decisions to perform CT head in mild TBI (e.g.
improving early identification of patients likely
to “talk and die”) and later biomarkers to predict
prolonged complications or to monitor TBI
recovery.
Numerous candidate biomarkers have proven
prognostic value with TBI outcome, such as: glial
fibrillary acidic protein (GFPA; glial cell injury),
unbiquitin C-terminal hydrolase-L1 (UCH-L1;
neuronal cell body injury), SBDP150/SBDP145
(spectrin breakdown products; axon and presynaptic terminal necrosis), S100B (CalciumBinding Protein B; elevated blood and urine
levels in glial injury), and NSE (neuron-specific
enolase; neuronal injury). However, they generally show low specificity or sensitivity when used
individually hence combining biomarkers into a
screening panel may provide more information
than individual biomarkers (e.g. GFAP/UCHL1, NSE/S100B)

In future, early biomarkers may facilitate deci- sions to perform CT head in mild TBI (e.g. improving early identification of patients likely to “talk and die”) and later biomarkers to predict prolonged complications or to monitor TBI recovery.
Numerous candidate biomarkers have proven prognostic value with TBI outcome, such as: glial fibrillary acidic protein (GFPA; glial cell injury), unbiquitin C-terminal hydrolase-L1 (UCH-L1; neuronal cell body injury), SBDP150/SBDP145 (spectrin breakdown products; axon and presyn- aptic terminal necrosis), S100B (Calcium- Binding Protein B; elevated blood and urine levels in glial injury), and NSE (neuron-specific enolase; neuronal injury). However, they gener- ally show low specificity or sensitivity when used individually hence combining biomarkers into a screening panel may provide more information than individual biomarkers (e.g. GFAP/UCH- L1, NSE/S100B).

How well did you know this?
1
Not at all
2
3
4
5
Perfectly
11
Q

Which one of the following is the LEAST appropriate indication for placement of ICP monitor in TBI?
a. GCS 3-8 with abnormal CT scan
b. GCS 3-8 with normal CT scan age >40 years and SBP <90 mmHg
c. Postoperative period after removal of acute subdural hematoma
d. GCS of 3-8 and diffuse injury type III (Marshall CT classification)
e. Diffuse injury type II (Marshall CT classification)

A

e. Diffuse injury type II (Marshall CT
classification)

The principle of ICP monitoring is to maintain
adequate cerebral perfusion and oxygenation to
meet metabolic demands. Raised ICP reduces
CPP and CBF, which exacerbates secondary
injury. Several studies have shown that patients
with an ICP <20 mmHg have a reduced risk of
neurological deterioration, compared with higher
ICPs (ICP 25 mmHg). Mortality also increases
dramatically from 17% to 47% when an average
ICP >20 mmHg. However, Level I and level II
evidence suggesting ICP monitoring improves
outcome in TBI is lacking. In one randomized
trial of ICP versus clinical examination plus imaging, there was no significant improvement in survival between the study groups. Similar findings
have been confirmed by other studies that have
concluded that ICP-targeted/CPP-targeted
intensive care tends to prolong mechanical ventilation and increases therapy intensity without evidence of improved outcome. Nonetheless, ICP
monitoring is recommended for the management
of severe TBI (GCS 3-8) with abnormal CT scan,
and in patients with normal CT scan but >2 of
age >40 years, SBP <90 mmHg, decorticate posturing, decerebrate posturing. Indications in
those with moderate TBI are less clear, but
include: (a) Postoperative period after removal
of acute subdural hematoma or multiple cerebral
contusion. In these cases, sudden changes in ICP
could signal hemorrhages due to decompression
or reperfusion, new extra-axial collections or
worsening brain swelling. (b) GCS of 9-11 and
cerebral contusion (temporal or bifrontal) without surgical intervention. In these instances,
ICP monitoring can help recognize progression
of the contusions. (c) Diffuse injury type III (Marshall CT classification: cisterns compressed or
absent with MLS <5 mm, no high or mixed density lesion >25 ml). Due to the high probability
of intracranial hypertension and poor outcome,
ICP monitoring in these cases is indispensable.
(d) General anesthesia for emergency non-cranial
surgery (especially in the presence of conservatively treated intracranial lesions) due to loss
of clinical evaluation and potential effects of
anesthetics on cerebrovascular autoregulation.
(e) Concomitant severe chest trauma requiring
deep sedation, high PEEP levels, recruitment
maneuvers or prone ventilation which may cause
hypercapnia or impair cerebral venous return,
causing cerebral vasodilation and increased ICP.
(f) Concomitant intra-abdominal compartment
syndrome (associated with intracranial hypertension). (g) Prolonged traumatic shock (risk of
cerebral edema).

The principle of ICP monitoring is to maintain
adequate cerebral perfusion and oxygenation to meet metabolic demands. Raised ICP reduces CPP and CBF, which exacerbates secondary injury. Several studies have shown that patients with an ICP <20 mmHg have a reduced risk of neurological deterioration, compared with higher ICPs (ICP !25 mmHg). Mortality also increases dramatically from 17% to 47% when an average ICP >20 mmHg. However, Level I and level II evidence suggesting ICP monitoring improves outcome in TBI is lacking. In one randomized trial of ICP versus clinical examination plus imag- ing, there was no significant improvement in sur- vival between the study groups. Similar findings have been confirmed by other studies that have concluded that ICP-targeted/CPP-targeted intensive care tends to prolong mechanical venti- lation and increases therapy intensity without evi- dence of improved outcome. Nonetheless, ICP monitoring is recommended for the management of severe TBI (GCS 3-8) with abnormal CT scan, and in patients with normal CT scan but >2 of age >40 years, SBP <90 mmHg, decorticate pos- turing, decerebrate posturing. Indications in those with moderate TBI are less clear, but include: (a) Postoperative period after removal of acute subdural hematoma or multiple cerebral contusion. In these cases, sudden changes in ICP could signal hemorrhages due to decompression or reperfusion, new extra-axial collections or worsening brain swelling. (b) GCS of 9-11 and cerebral contusion (temporal or bifrontal) with- out surgical intervention. In these instances, ICP monitoring can help recognize progression of the contusions. (c) Diffuse injury type III (Mar- shall CT classification: cisterns compressed or absent with MLS <5 mm, no high or mixed den- sity lesion >25 ml). Due to the high probability of intracranial hypertension and poor outcome, ICP monitoring in these cases is indispensable. (d) General anesthesia for emergency non-cranial surgery (especially in the presence of conserva- tively treated intracranial lesions) due to loss of clinical evaluation and potential effects of anesthetics on cerebrovascular autoregulation. (e) Concomitant severe chest trauma requiring deep sedation, high PEEP levels, recruitment maneuvers or prone ventilation which may cause hypercapnia or impair cerebral venous return, causing cerebral vasodilation and increased ICP. (f) Concomitant intra-abdominal compartment syndrome (associated with intracranial hyperten- sion). (g) Prolonged traumatic shock (risk of cerebral edema).

How well did you know this?
1
Not at all
2
3
4
5
Perfectly
12
Q

A 17-year-old boy is struck on the right side of the head during a sports match. He is dazed initially and is taken off the field despite saying he feels fine. He becomes more sleepy over the subsequent 15min and an ambulance is called. In the emergency department his GCS is E2V3M5 with a sluggish right pupil and weakness on the left side of his body. CT head is shown. Which one of the following is LEAST likely to be an indication for surgery in this type of pathology?
a. Lesion volume greater than 30 cm3 with anisocoria and 20 mm maximal thickness
b. Lesion volume greater than 30 cm3
c. GCS 8 or less with evidence of anisocoria
d. Lesionvolume15cm3withanisocoriabut no midline shift
e. Lesion volume 35cm3 with atrophic brain without midline shift

A

e. Lesion volume 35cm3 with atrophic brain
without midline shift

The clinical scenario is of a right frontotemporal
EDH with a swirling appearance on imaging due
to leakage of serum from clot or active bleeding.
EDH is seen in 2.7-4% of traumatic brain injury,
and 9% of severe TBI. Mortality approximates
10%. Peak incidence of EDH is in the second
decade, and the mean age of patients with EDH
is between 20 and 30 years of age. EDH can result
from injury to the middle meningeal artery
(90%), the middle meningeal vein, the diploic
veins, or the venous sinuses. Venous epidural less
common—usually post fossa, usually pediatric,
can extend across tentorium to be both supra
and infratentorial (transverse/sigmoid sinus),
paramedian/vertex (superior sagittal), middle cranial fossa floor (sphenoparietal sinus). Injury can
result in pseudoaneurysm of meningeal artery or
a dural AV fistula if both artery and vein lacerated. Presentation is with coma (one third to a
half), lucid interval (less than a half) and the
remainder remain conscious. Pupillary abnormalities are present in approximately 20-40%.
The hematoma volume can be estimated quickly
from the head CT scan by using the formula
ABC/2, which approximates the volume of
an ellipsoid. On the CT slice with the largest area
of hemorrhage, A is the greatest hemorrhage
diameter and B is the largest diameter perpendicular to A. Giving each slice a value of 1 or 0.5 if
the area is >75% or 25-75% of the index slice
used to measure A and B, their sum is multiplied
by slice thickness in cm to give C. An epidural
hematoma (EDH) greater than 30 cm3 should
be surgically evacuated regardless of the patient’s
Glasgow Coma Scale (GCS) score. An EDH less
than 30 cm3 and with less than a 15-mm thickness
and with less than a 5-mm midline shift (MLS) in
patients with a GCS score greater than 8 without
focal deficit can be managed non-operatively with
serial computed tomographic (CT) scanning and
close neurological observation in a neurosurgical
center, but enlargement occurs in 20% of cases. It
is strongly recommended that patients with an
acute EDH in coma (GCS score <9) with anisocoria undergo surgical evacuation as soon as possible (i.e. within 1 h). Bilateral EDH is
comparatively uncommon entity, accounting for
approximately 2-5% of adults with extradural hematoma. Kett-White and Martin classified
such cases into two distinct groups: patients with
bilateral but separate extradural hematoma
located in the convexities, resulting from dura
being stripped from the skull independently on
both sides, either simultaneously or sequentially;
and, more rarely, patients with bilateral extradural hematoma straddling the midline, resulting
from injury to the sagittal sinus. Although all
identified cases with bilateral extradural hematoma resulting from sagittal sinus injury underwent urgent surgical evacuation, the operative
technique varied regarding preserve the midline
skull vault and use dural tenting sutures, or—in
selected cases—expose the sagittal sinus and
attempt primary repair

The clinical scenario is of a right frontotemporal EDH with a swirling appearance on imaging due to leakage of serum from clot or active bleeding. EDH is seen in 2.7-4% of traumatic brain injury, and 9% of severe TBI. Mortality approximates 10%. Peak incidence of EDH is in the second decade, and the mean age of patients with EDH is between 20 and 30 years of age. EDH can result from injury to the middle meningeal artery (90%), the middle meningeal vein, the diploic veins, or the venous sinuses. Venous epidural less common—usually post fossa, usually pediatric, can extend across tentorium to be both supra and infratentorial (transverse/sigmoid sinus), paramedian/vertex (superior sagittal), middle cra- nial fossa floor (sphenoparietal sinus). Injury can result in pseudoaneurysm of meningeal artery or a dural AV fistula if both artery and vein lacer- ated. Presentation is with coma (one third to a half), lucid interval (less than a half) and the remainder remain conscious. Pupillary abnor- malities are present in approximately 20-40%. The hematoma volume can be estimated quickly from the head CT scan by using the formula ABC/2, which approximates the volume of an ellipsoid. On the CT slice with the largest area of hemorrhage, A is the greatest hemorrhage diameter and B is the largest diameter perpendic- ular to A. Giving each slice a value of 1 or 0.5 if the area is >75% or 25-75% of the index slice used to measure A and B, their sum is multiplied by slice thickness in cm to give C. An epidural hematoma (EDH) greater than 30 cm3 should be surgically evacuated regardless of the patient’s Glasgow Coma Scale (GCS) score. An EDH less than 30 cm3 and with less than a 15-mm thickness and with less than a 5-mm midline shift (MLS) in patients with a GCS score greater than 8 without focal deficit can be managed non-operatively with serial computed tomographic (CT) scanning and close neurological observation in a neurosurgical center, but enlargement occurs in 20% of cases. It is strongly recommended that patients with an acute EDH in coma (GCS score <9) with aniso- coria undergo surgical evacuation as soon as pos- sible (i.e. within 1h). Bilateral EDH is comparatively uncommon entity, accounting for approximately 2-5% of adults with extradural hematoma. Kett-White and Martin classified such cases into two distinct groups: patients with bilateral but separate extradural hematoma located in the convexities, resulting from dura being stripped from the skull independently on both sides, either simultaneously or sequentially; and, more rarely, patients with bilateral extra- dural hematoma straddling the midline, resulting from injury to the sagittal sinus. Although all identified cases with bilateral extradural hema- toma resulting from sagittal sinus injury under- went urgent surgical evacuation, the operative technique varied regarding preserve the midline skull vault and use dural tenting sutures, or—in selected cases—expose the sagittal sinus and attempt primary repair.

How well did you know this?
1
Not at all
2
3
4
5
Perfectly
13
Q

A27-year-oldpresentstoEDafteranassault. His GCS is 15/15 but he has evidence of facial fractures involving the frontal sinus and evidence of some CSF rhinorrhea. He is admitted for observation and initial conser- vative management of CSF leak. On D3 post injury, he developed three episodes of vomit- ing and became drowsy. On examination he was obtunded and lethargic, but arousable. His BP was 140/90 mmHg, heart rate 59/ min, and respiratory rate 20/min and main- tained a saturation of 92% on room air. Examination revealed a dilated right pupil whereas the rest of neurological and systemic examination was normal. CT head was repeated (shown). Which one of the follow- ing is most appropriate acute management?
a. High flow oxygen
b. Burr hole decompression
c. Cranialization of the frontal sinus
d. Decompressive craniectomy
e. Minicraniotomy and excision of membranes

A

b. Burr hole decompression

Pneumocephalus is the presence of intracranial
air, which invariably resolves spontaneously or
with conservative treatment. However, clinical
deterioration can occur in tension pneumocephalus where a progressive accumulation of intracranial air which cannot escape (ball valve
mechanism) exerts mass effect on the brain and
can lead to coma, herniation and death. Tension
PC is commonly caused by intra or extra-cranial
surgeries like drainage of chronic subdural
hematoma, craniofacial surgery, otorhinolaryngological procedures, shunt operations/CSF
drainage, trauma (fractures through skull base
involving air sinuses), meningitis/otitis, anesthesia
(spinal, ventilation, nitric oxide induced), and
tumors. Accumulation of trapped air in subdural
and interhemispheric space bilaterally, is seen as
hypodense collections causing compression and
separation of the frontal lobes (Mount Fuji sign).
Emergency management is by decompression,
usually by opening up previous burr hole (if
present) or making a new one. Definitive
management will involve identifying the site of
air entry and intracranial or extracranial repair

Pneumocephalus is the presence of intracranial air, which invariably resolves spontaneously or with conservative treatment. However, clinical deterioration can occur in tension pneumocepha- lus where a progressive accumulation of intracra- nial air which cannot escape (ball valve mechanism) exerts mass effect on the brain and can lead to coma, herniation and death. Tension PC is commonly caused by intra or extra-cranial surgeries like drainage of chronic subdural hematoma, craniofacial surgery, otorhinolaryn- gological procedures, shunt operations/CSF drainage, trauma (fractures through skull base involving air sinuses), meningitis/otitis, anesthesia (spinal, ventilation, nitric oxide induced), and tumors. Accumulation of trapped air in subdural and interhemispheric space bilaterally, is seen as hypodense collections causing compression and separation of the frontal lobes (Mount Fuji sign). Emergency management is by decompression, usually by opening up previous burr hole (if present) or making a new one. Definitive management will involve identifying the site of air entry and intracranial or extracranial repair.

How well did you know this?
1
Not at all
2
3
4
5
Perfectly
14
Q

A 57-year-old patient presents with head injury after falling backwards and sudden onset of agitation. Two hours later her blood pressure is 220/110 mmHg, her heart rate is 39 beats per minute, and her consciousness is fluctuating. She is intubated and ventilated. CT is shown. Regarding this type of lesion and its location, which one of the following is LEAST likely to be an indication for surgical evacuation?
a. Neurological dysfunction or deterioration referable to the lesion
b. Distortion, dislocation, or obliteration of the fourth ventricle
c. Compression or loss of visualization of the temporal horns
d. Presence of obstructive hydrocephalus.
e. Underlying depressed skull fracture

A

c—Compression or loss of visualization of
the temporal horns

The clinical scenario suggests a Cushing’s reflex
secondary to an expanding right sided posterior
fossa extradural hematoma, most probably due
to transverse sinus laceration, and evidence of
4th ventricular effacement and obstructive
hydrocephalus. Upward herniation of cerebellar
vermis through the tentorial incisura is also seen.
Indications for surgery include neurological
dysfunction or deterioration referable to the
lesion, or presence of mass effect: distortion, dislocation, or obliteration of the fourth ventricle;
compression or loss of visualization of the basal
cisterns, or the presence of obstructive hydrocephalus. Evacuation via suboccipital craniectomy should be performed as soon as possible
because these patients can deteriorate rapidly,
thus, worsening their prognosis. Patients with
lesions and no significant mass effect on CT scan
and without signs of neurological dysfunction
may be managed by close observation and serial
imaging.

The clinical scenario suggests a Cushing’s reflex secondary to an expanding right sided posterior fossa extradural hematoma, most probably due to transverse sinus laceration, and evidence of 4th ventricular effacement and obstructive hydrocephalus. Upward herniation of cerebellar vermis through the tentorial incisura is also seen. Indications for surgery include neurological dysfunction or deterioration referable to the lesion, or presence of mass effect: distortion, dis- location, or obliteration of the fourth ventricle; compression or loss of visualization of the basal cisterns, or the presence of obstructive hydro- cephalus. Evacuation via suboccipital craniect- omy should be performed as soon as possible because these patients can deteriorate rapidly, thus, worsening their prognosis. Patients with lesions and no significant mass effect on CT scan and without signs of neurological dysfunction may be managed by close observation and serial imaging.

How well did you know this?
1
Not at all
2
3
4
5
Perfectly
15
Q

A 19-year-old female is involved in a high speed motor vehicle accident and sustained facial injuries from hitting the dashboard. She was GCS 4/15 at the scene and was intubated for transfer. Her pupils are reactive to light. CT head was performed as part of the trauma protocol but did not show any mass lesions or fractures. As this was an isolated head injury a decision was made to wean sedation and assess neurology, but she remained unresponsive. CT is shown below. These are most likely indicative of which one of the following?
a. Subarachnoid hemorrhage
b. Diffuse axonal injury
c. Cortical contusion
d. Global hypoxic brain damage
e. Cerebral venous sinus thrombosis

A

b. Diffuse axonal injury

Diffuse axonal injury is the most common cause
of coma in the head-injured patient without an
intracranial mass lesion. It is characterized pathologically by diffusely spread axonal swellings
affecting the white matter, corpus callosum, and
upper brainstem. These foci are usually hemorrhagic. The etiology is thought to be due to
shearing forces on axons in certain susceptible
regions of the brain, notably those that are particularly vulnerable to rotational forces, such as the
subcortical white matter, corpus callosum, and
upper brainstem. On MRI the spots will appear
as T2 bright lesions and appear dark on GRE.
Classically, DAI has been considered to be a
major neuropathological feature of severe TBI,
but most experts now agree that almost all TBI is usually accompanied by some degree of DAI
and brain volume loss. There have been no
evidence-based effective treatments specifically
for DAI, hence avoidance of secondary brain
injury, and medial and/or surgical management
of ICP are the mainstay

Diffuse axonal injury is the most common cause of coma in the head-injured patient without an intracranial mass lesion. It is characterized path- ologically by diffusely spread axonal swellings affecting the white matter, corpus callosum, and upper brainstem. These foci are usually hemor- rhagic. The etiology is thought to be due to shearing forces on axons in certain susceptible regions of the brain, notably those that are partic- ularly vulnerable to rotational forces, such as the subcortical white matter, corpus callosum, and upper brainstem. On MRI the spots will appear as T2 bright lesions and appear dark on GRE. Classically, DAI has been considered to be a major neuropathological feature of severe TBI, but most experts now agree that almost all TBI is usually accompanied by some degree of DAI
and brain volume loss. There have been no evidence-based effective treatments specifically for DAI, hence avoidance of secondary brain injury, and medial and/or surgical management of ICP are the mainstay.

How well did you know this?
1
Not at all
2
3
4
5
Perfectly
16
Q

Which one of the following statements regarding chronic subdural hematomas is LEAST accurate?
a. They form due to separation of the dural border cell layer
b. They may arise secondary to acute sub- dural hematoma with fails to be resorbed
c. They may arise secondary to repeated hemorrhage into a subdural hygroma
d. They are prone to forming membranes with a tendency to bleed
e. They are more common in females than males at all ages

A

e. They are more common in females than
males at all ages

A chronic subdural hematoma (CSDH) is a collection of blood breakdown products in the subdural space. Estimates of the incidence of CSDH
range from 8.2 to 14.0 per 100,000 person-years.
Presentation is predominantly in the seventh
decade onwards and is more frequent in males
(approximately 3:1 male to female ratio across
all age groups). CSDH arises at the dural border
cell layer (between the dura mater and arachnoid
mater) which is prone to separation creating a
“subdural space,” particularly in those with cerebral atrophy (e.g. elderly, alcoholics). Minor precipitant trauma can generate CSDH either from
the incomplete breakdown/reabsorption of
ASDH or recurrent microhemorrhage into subdural hygromas. ASDH may occur due to tearing
of the bridging veins that traverse the dural border cell layer or, less commonly, tearing of cortical arteries or veins. In contrast, a subdural
hygroma (CSF collection), is caused by splitting
of the dural border cell layer at points of tension
between the dura mater and arachnoid mater.
Neovascularization occurs in the newly formed
subdural space and hemorrhage from these new
vessels leads to the formation of a CSDH. In both
cases, the opening of the dural border cell layer
into a subdural space triggers a reparatory
response which either causes hematoma/
hygroma resolution or hematoma enlargement.
The latter is thought to be due to a localized
inflammatory reaction, which results in hyperfibrinolysis of the clot and production of angiogenic factors that promote neovascularization
and further bleeding from fragile capillaries. Formation of neomembranes is one of the main features of CSDH—the inner (visceral) membrane is
less vascular and usually thinner than the outer
(parietal) membrane. Risk factors for CSDH
include advancing age, a history of falls, minor
head injury, use of anticoagulants or antiplatelet
drugs, bleeding diatheses, alcohol (contributing
to globalized brain atrophy, increased risk of falls,
and hepatogenic coagulopathy), epilepsy, intracranial hypotension, and hemodialysis.

A chronic subdural hematoma (CSDH) is a col- lection of blood breakdown products in the sub- dural space. Estimates of the incidence of CSDH range from 8.2 to 14.0 per 100,000 person-years. Presentation is predominantly in the seventh decade onwards and is more frequent in males (approximately 3:1 male to female ratio across all age groups). CSDH arises at the dural border cell layer (between the dura mater and arachnoid mater) which is prone to separation creating a “subdural space,” particularly in those with cere- bral atrophy (e.g. elderly, alcoholics). Minor pre- cipitant trauma can generate CSDH either from the incomplete breakdown/reabsorption of ASDH or recurrent microhemorrhage into sub- dural hygromas. ASDH may occur due to tearing of the bridging veins that traverse the dural bor- der cell layer or, less commonly, tearing of corti- cal arteries or veins. In contrast, a subdural hygroma (CSF collection), is caused by splitting of the dural border cell layer at points of tension between the dura mater and arachnoid mater. Neovascularization occurs in the newly formed subdural space and hemorrhage from these new vessels leads to the formation of a CSDH. In both cases, the opening of the dural border cell layer into a subdural space triggers a reparatory response which either causes hematoma/ hygroma resolution or hematoma enlargement. The latter is thought to be due to a localized inflammatory reaction, which results in hyperfi- brinolysis of the clot and production of angio- genic factors that promote neovascularization and further bleeding from fragile capillaries. For- mation of neomembranes is one of the main fea- tures of CSDH—the inner (visceral) membrane is less vascular and usually thinner than the outer (parietal) membrane. Risk factors for CSDH include advancing age, a history of falls, minor head injury, use of anticoagulants or antiplatelet drugs, bleeding diatheses, alcohol (contributing to globalized brain atrophy, increased risk of falls, and hepatogenic coagulopathy), epilepsy, intra- cranial hypotension, and hemodialysis.

17
Q

A 72-year-old male develops a worsening headache over the course of 3 weeks. There is no relevant past medical history, except for a minor head injury 6 weeks ago. On examination there is no focal deficit. CT head is shown. In the UK, which one of the following is appropriate next management?
a. Twist drill craniotomy
b. Single burr hole with or without subdural drain
c. Humanprothrombincomplexconcentrate
d. Dexamethasone 2 mg twice daily
e. Minicraniotomy and excision of membranes

A

b. Single burr hole with or without
subdural drain
Medical management of Subacute SDH or
CSDH generally involves correction of bleeding
disorders/anticoagulation and any interventions
required to improve anesthetic fitness (e.g. treat
infections, optimize gas exchange, etc.). Steroids
may be useful as adjunctive therapy preoperatively or postoperatively, or as monotherapy as
an alternative to surgical intervention (in small,
mildly symptomatic CSDH), but their role
remains ill-defined in the absence of level I evidence. Preoperative antiepileptic prophylaxis
has been shown to affect postoperative seizure
rate but not discharge outcome, hence is not particularly advocated. In patients with mild or no
symptoms and relatively small collections, or
moribund patients with poor baseline function
may both be managed non-operatively. However,
surgical treatment of symptomatic CSDH results
in rapid improvement of symptoms, and produces
a favorable outcome in over 80% of patients.
Coupled with relatively low surgical risk,
surgical evacuation is currently the mainstay of
management for symptomatic patients. Three
primary surgical techniques are used: twist drill
craniostomy (TDC) involving small openings
(<10 mm) made using a twist drill, burr hole craniostomy (BHC; most popular) involving openings of 10-30 mm, and craniotomy involving
larger openings. The BHC technique involves
the drilling of burr holes over the cerebral convexity followed by durotomy, usually under general anesthetic. Insertion of a subdural drain in
the anterior hole connected to a closed drainage
system for approximately 48 h decreased CSDH
recurrence after BHC from 24.3% to 9.3%
(P¼0.003) and 6-month mortality from 18.1%
to 9.6% (P¼0.042). Twist drill craniostomy can
be performed under local anesthesia either at
the bedside or in the operating theatre, making
it an attractive option for the elderly patient with
multiple comorbidities. A recent modification to
the original technique involves the insertion of a
hollow screw to set up a closed drainage system.
This technique, unlike the traditional TDC technique, does not require the blind insertion of a
catheter in the subdural space, thereby minimizing the risks of brain laceration and bleeding from
cortical vessels. Craniotomy involves general
anesthesia and is commonly reserved for recurrent CSDH with extensive organization and membrane formation (preventing brain reexpansion) or primary evacuation of a CSDH that
has a substantial acute component. Systematic
reviews comparing the three techniques have
generally found either no difference between
TDC and BHC, or lower recurrence rates with
BHC and craniotomy, higher morbidity with craniotomy (and in one review BHC), and a higher
mortality with craniotomy

Medical management of Subacute SDH or CSDH generally involves correction of bleeding disorders/anticoagulation and any interventions required to improve anesthetic fitness (e.g. treat infections, optimize gas exchange, etc.). Steroids may be useful as adjunctive therapy preopera- tively or postoperatively, or as monotherapy as an alternative to surgical intervention (in small, mildly symptomatic CSDH), but their role remains ill-defined in the absence of level I evi- dence. Preoperative antiepileptic prophylaxis has been shown to affect postoperative seizure rate but not discharge outcome, hence is not par- ticularly advocated. In patients with mild or no symptoms and relatively small collections, or moribund patients with poor baseline function may both be managed non-operatively. However, surgical treatment of symptomatic CSDH results in rapid improvement of symptoms, and produces a favorable outcome in over 80% of patients. Coupled with relatively low surgical risk, surgical evacuation is currently the mainstay of management for symptomatic patients. Three primary surgical techniques are used: twist drill craniostomy (TDC) involving small openings (<10 mm) made using a twist drill, burr hole cra- niostomy (BHC; most popular) involving open- ings of 10-30mm, and craniotomy involving larger openings. The BHC technique involves the drilling of burr holes over the cerebral con- vexity followed by durotomy, usually under gen- eral anesthetic. Insertion of a subdural drain in the anterior hole connected to a closed drainage system for approximately 48 h decreased CSDH recurrence after BHC from 24.3% to 9.3% (P1⁄40.003) and 6-month mortality from 18.1% to 9.6% (P1⁄40.042). Twist drill craniostomy can be performed under local anesthesia either at the bedside or in the operating theatre, making it an attractive option for the elderly patient with multiple comorbidities. A recent modification to the original technique involves the insertion of a hollow screw to set up a closed drainage system. This technique, unlike the traditional TDC tech- nique, does not require the blind insertion of a catheter in the subdural space, thereby minimiz- ing the risks of brain laceration and bleeding from cortical vessels. Craniotomy involves general anesthesia and is commonly reserved for recur- rent CSDH with extensive organization and membrane formation (preventing brain re- expansion) or primary evacuation of a CSDH that has a substantial acute component. Systematic reviews comparing the three techniques have generally found either no difference between TDC and BHC, or lower recurrence rates with BHC and craniotomy, higher morbidity with cra- niotomy (and in one review BHC), and a higher mortality with craniotomy.

18
Q

A 29-year-old is assaulted and is GCS E3V4M5 at the scene. On examination there is significant bruising to the right side of his head with a 5-cm laceration and does not appear to have any focal neurological deficit. He is awaiting a CT scan when you are told he is less responsive. On examination, he is GCS E2V3M4, his right pupil is dilated and on application of painful stimulus appears to only to be moving his left side. Which one of the following is most likely to be responsible for these findings?
a. Transalar (transsphenoidal) herniation
b. Unilateral uncal herniation
c. Ascending transtentorial herniation
d. Tonsillarherniationthroughtheforamen magnum
e. Subfalcine herniation

A

b. Unilateral uncal herniation

Subfalcine herniation is the commonest type of
herniation patter as it is measured by midline shift
of the septum pellucidum, and reflects cingulate
gyrus, anterior cerebral artery, internal cerebral
vein and lateral ventricle compression. Complications of subfalcine herniation are contralateral
lateral ventricle hydrocephalus due to obstruction
of the foramen of Monro, and ipsilateral ACA
compression against the falx resulting in infarction and contralateral leg weakness. Following
subfalcine herniation, descending transtentorial
herniation is the second most common type of
herniation syndrome and may occur unilaterally
or bilaterally. With increasing supratentorial
mass effect, the uncus of the anteromedial temporal lobe is driven medially to efface the suprasellar
cistern (uncal herniation). Medial shift of the hippocampus and parahippocampal gyrus then follows with subsequent effacement of the
mesencephalic and quadrigeminal cisterns and
compression of the midbrain. Kernohan’s notch
phenomenon is an uncommon but important scenario described by this case where a unilateral
hemispheric expanding mass lesion causing ipsilateral uncal herniation (descending transtentorial herniation; associated with 3rd nerve palsy)
results in compression of the contralateral cerebral peduncle against the tentorium cerebelli creating a “notch” in it. This leads to the confusing
picture of hemiparesis on the same side as the
mass lesion (false localizing sign). By contrast diffuse brain swelling (e.g. hypoxic brain injury)
is likely to cause a more symmetrical uncal herniation without this phenomenon. With midbrain
caudal compression, there is progressive narrowing of the normal angle (90°) between the ventral midbrain and pons with subsequent
compression of basilar artery perforators and
potentiation for secondary midbrain hemorrhagic infarct—Duret hemorrhage. Ascending
transtentorial cerebellar herniation through the
tentorial notch due to a mass lesion in the posterior fossa causes rapid loss of consciousness, noncommunicating hydrocephalus and PCA/SCA
territory infarction. Descending transalar herniation occurs as a result of frontal lobe mass effect
with posterior and inferior displacement of the
posterior aspect of the frontal lobe orbital surface
over the sphenoid wing and MCA compression.
Ascending transalar herniation is produced by
middle cranial fossa or temporal lobemass effect
resulting in displacement of the temporal lobe
superiorly and anteriorly across the sphenoid
ridge causing compression of the ICA. Tonsillar
cerebellar herniation though the foramen magnum may be chronic/congenital (e.g. Chiari I
malformation), due to primary posterior fossa
mass lesions or secondary to descending transtentorial herniation from a supratentorial mass
lesion; compression of the brainstem against the
clivus results in compression of respiratory and
cardiac centers

Subfalcine herniation is the commonest type of herniation patter as it is measured by midline shift of the septum pellucidum, and reflects cingulate gyrus, anterior cerebral artery, internal cerebral vein and lateral ventricle compression. Complica- tions of subfalcine herniation are contralateral lateral ventricle hydrocephalus due to obstruction of the foramen of Monro, and ipsilateral ACA compression against the falx resulting in infarc- tion and contralateral leg weakness. Following subfalcine herniation, descending transtentorial herniation is the second most common type of herniation syndrome and may occur unilaterally or bilaterally. With increasing supratentorial mass effect, the uncus of the anteromedial tempo- ral lobe is driven medially to efface the suprasellar cistern (uncal herniation). Medial shift of the hip- pocampus and parahippocampal gyrus then fol- lows with subsequent effacement of the mesencephalic and quadrigeminal cisterns and compression of the midbrain. Kernohan’s notch phenomenon is an uncommon but important sce- nario described by this case where a unilateral hemispheric expanding mass lesion causing ipsi- lateral uncal herniation (descending transtentor- ial herniation; associated with 3rd nerve palsy) results in compression of the contralateral cere- bral peduncle against the tentorium cerebelli cre- ating a “notch” in it. This leads to the confusing picture of hemiparesis on the same side as the mass lesion (false localizing sign). By contrast,
diffuse brain swelling (e.g. hypoxic brain injury) is likely to cause a more symmetrical uncal herni- ation without this phenomenon. With midbrain caudal compression, there is progressive narrow- ing of the normal angle ($90°) between the ven- tral midbrain and pons with subsequent compression of basilar artery perforators and potentiation for secondary midbrain hemor- rhagic infarct—Duret hemorrhage. Ascending transtentorial cerebellar herniation through the tentorial notch due to a mass lesion in the poste- rior fossa causes rapid loss of consciousness, non- communicating hydrocephalus and PCA/SCA territory infarction. Descending transalar hernia- tion occurs as a result of frontal lobe mass effect with posterior and inferior displacement of the posterior aspect of the frontal lobe orbital surface over the sphenoid wing and MCA compression. Ascending transalar herniation is produced by middle cranial fossa or temporal lobemass effect resulting in displacement of the temporal lobe superiorly and anteriorly across the sphenoid ridge causing compression of the ICA. Tonsillar cerebellar herniation though the foramen mag- num may be chronic/congenital (e.g. Chiari I malformation), due to primary posterior fossa mass lesions or secondary to descending transten- torial herniation from a supratentorial mass lesion; compression of the brainstem against the clivus results in compression of respiratory and cardiac centers.

19
Q

A 45-year-old is involved in a high speed road traffic accident and was GCS E2V3M4 at the scene. He was intubated and ventilated and transferred to a trauma center. En route, his left pupil became sluggish and mannitol was administered. CT head scan is shown. Given the scenario and type of injury shown, which one of the following is the LEAST appropriate indication for surgery according to current guidelines?
a. Thickness greater than 10 mm
b. Midline shift greater than 5 mm
c. GCS score less than 9, with or without ICP >20 mmHg
d. Documented drop in GCS by 2 or more points since injury
e. Asymmetric or fixed and dilated pupils

A

c. GCS score less than 9, with or without
ICP >20 mmHg

Acute SDH complicates 11% of all TBI, and 10-
30% of severe TBI. Most SDH are caused by
motor vehicle-related accidents (MVA), falls,
and assaults. The elderly population is particularly susceptible due to increased fragility of vessel walls, falls and greater use of antithrombotic
and anticoagulants agents. Between 37% and
80% of patients with acute SDH present with initial GCS scores of 8 or less. A lucid interval has
been described in 12-38% of patients before
admission but there is no conclusive evidence that
this correlates with outcome. The definition of
lucid interval is vague. Authors interpret the lucid
interval differently and analysis of its frequency
requires documentation during the prehospital
phase. Pupillary abnormalities are observed in
30-50% of patients on admission or before surgery. Compared with EDH, the degree of underlying brain damage associated with ASDH is
more severe, and mortality rates are greater
especially in older patients with poor initial GCS, and other associated brain or systemic injuries. Studies looking at patients from all age groups
with GCS scores between 3 and 15 with SDH
requiring surgery quote mortality rates between
40 and 60%. Mortality among patients presenting
to the hospital in coma with subsequent surgical
evacuation is between 57% and 68%. Only 30-
40% of SDH requiring surgery are isolated
lesions. In the majority of cases, the SDH is associated with other intracranial and extracranial injuries. An acute subdural hematoma (SDH) with a
thickness greater than 10 mm or a midline shift
greater than 5 mm on computed tomographic
(CT) scan should be surgically evacuated, regardless of the patient’s Glasgow Coma Scale (GCS)
score. All patients with acute SDH in coma
(GCS score less than 9) should undergo intracranial pressure (ICP) monitoring. A comatose
patient (GCS score less than 9) with an SDH less
than 10 mm thick and a midline shift less than
5 mm should undergo surgical evacuation of the
lesion if the GCS score decreased between the
time of injury and hospital admission by 2 or more
points on the GCS and/or the patient presents
with asymmetric or fixed and dilated pupils and/
or the ICP exceeds 20 mmHg. In patients with
acute SDH and indications for surgery, surgical
evacuation should be performed as soon as possible, with or without bone flap removal and duraplasty. Patients with less severe ASDH can be
monitored clinically; after 7-10 days an ASDH
may liquefy, to become drainable with burr holes,
thus avoiding the major morbidity of craniotomy

Acute SDH complicates 11% of all TBI, and 10- 30% of severe TBI. Most SDH are caused by motor vehicle-related accidents (MVA), falls, and assaults. The elderly population is particu- larly susceptible due to increased fragility of ves- sel walls, falls and greater use of antithrombotic and anticoagulants agents. Between 37% and 80% of patients with acute SDH present with ini- tial GCS scores of 8 or less. A lucid interval has been described in 12-38% of patients before admission but there is no conclusive evidence that this correlates with outcome. The definition of lucid interval is vague. Authors interpret the lucid interval differently and analysis of its frequency requires documentation during the prehospital phase. Pupillary abnormalities are observed in 30-50% of patients on admission or before sur- gery. Compared with EDH, the degree of under- lying brain damage associated with ASDH is more severe, and mortality rates are greater especially in older patients with poor initial GCS, and other associated brain or systemic inju- ries. Studies looking at patients from all age groups with GCS scores between 3 and 15 with SDH requiring surgery quote mortality rates between 40 and 60%. Mortality among patients presenting to the hospital in coma with subsequent surgical evacuation is between 57% and 68%. Only 30- 40% of SDH requiring surgery are isolated lesions. In the majority of cases, the SDH is asso- ciated with other intracranial and extracranial inju- ries. An acute subdural hematoma (SDH) with a thickness greater than 10 mm or a midline shift greater than 5mm on computed tomographic (CT) scan should be surgically evacuated, regard- less of the patient’s Glasgow Coma Scale (GCS) score. All patients with acute SDH in coma (GCS score less than 9) should undergo intracra- nial pressure (ICP) monitoring. A comatose patient (GCS score less than 9) with an SDH less than 10 mm thick and a midline shift less than 5 mm should undergo surgical evacuation of the lesion if the GCS score decreased between the time of injury and hospital admission by 2 or more points on the GCS and/or the patient presents with asymmetric or fixed and dilated pupils and/ or the ICP exceeds 20 mmHg. In patients with acute SDH and indications for surgery, surgical evacuation should be performed as soon as possi- ble, with or without bone flap removal and dura- plasty. Patients with less severe ASDH can be monitored clinically; after 7-10 days an ASDH may liquefy, to become drainable with burr holes, thus avoiding the major morbidity of craniotomy.

20
Q

A 23-year-old female sustains a head injury after accidentally falling from a motorcycle at low speed. She is drowsy at scene and say- ing inappropriate words, but obeys com- mands and remained in this state for several hours in the emergency department. CT head showed a small amount of convexity traumatic subarachnoid blood. Which one of the following best reflects her risk of devel- oping post-traumatic epilepsy?
a. <1%
b. 5%
c. 10%
d. 15%
e. 20%

A

b. 5%

Post-traumatic seizures are classified into early
(<1 week post-injury) and late (>1 week postinjury). Generally children are more likely to
develop early seizures, whereas adults are more
likely to develop post-traumatic epilepsy. Overall,
the rate of post-traumatic epilepsy after TBI is
approximately 2% over the subsequent 10 years.
However, frequency increases with severity of
TBI: the risk is only marginally higher than the
general population in mild TBI, less than 5% in
moderate TBI and affects 10-15% of patients
after severe TBI. Short-term anti-epileptic prophylaxis for high-risk cases (e.g. cortical contusion, depressed skull fracture) as it has been
shown to reduce the incidence of early seizures;
however, there is no evidence supporting the
use of long-term prophylaxis to prevent posttraumatic epilepsy.

Post-traumatic seizures are classified into early (<1 week post-injury) and late (>1 week post- injury). Generally children are more likely to develop early seizures, whereas adults are more likely to develop post-traumatic epilepsy. Overall, the rate of post-traumatic epilepsy after TBI is approximately 2% over the subsequent 10 years. However, frequency increases with severity of TBI: the risk is only marginally higher than the general population in mild TBI, less than 5% in moderate TBI and affects 10-15% of patients after severe TBI. Short-term anti-epileptic pro- phylaxis for high-risk cases (e.g. cortical
contusion, depressed skull fracture) as it has been shown to reduce the incidence of early seizures; however, there is no evidence supporting the use of long-term prophylaxis to prevent post- traumatic epilepsy.

21
Q

Which one of the following statements regarding the lesion shown is LEAST accurate?
a. Pattern represents about 20% of cases of moderate TBI cerebral contusions
b. Due to acceleration/deceleration injury
c. Associated with ascending transalar herniation
d.Decompressive craniectomy would require cutting the anterior falx
e. Cerebral edema usually peaks between day 5 and 10

A

c. Associated with ascending transalar herniation

Bifrontal contusions are present in about 20% of
cases of moderate TBI with cerebral contusion
and are due to acceleration/deceleration forces
pushing the inferior frontal lobes and temporal
pole against the irregular skull base. Hemorrhagic swelling of these contusions may disrupt
the median forebrain bundle, gyrus rectus, and
anterior hypothalamic nuclei. These structures
are involved in behavior control and are associated with changes in personality, volition, motivation, judgment, and social interactions. When
these contusions swell, the brain is displaced posteriorly (descending transalar herniation) and
abrupt deterioration because of descent of the
brain stem into the posterior fossa with stretching
and deformity of the small perforating blood vessels of the basilar artery with risk of death due to
respiratory arrest, sudden coma, and autonomic
changes. These lesions are also very often associated with disturbances of sodium and water
metabolism (e.g. diabetes insipidus or SIADH).
Aggressive surgical resection of these contusions
may worsen the late neurological deficit and neuropsychological consequences. Surgical decompression requires bifrontal decompressive
craniotomy and cutting of the falx and sagittal
sinus on the frontal cranial fossa. This is a major
procedure, caries hemorrhage risk, and requires
delayed cranioplasty 2-3 months later. The risk
of this surgical procedure must, therefore, be balanced against risk of death due to herniation. The
timing of deterioration is variable, but brain
edema will usually peak around the 5th to the
10th day, with resolution of the swelling after this
time hence observation for up to 2 weeks with
serial CT scanning every 2-3 days is necessary
to exclude progression of the lesions. When the
patients are unable to obey commands, are very
restless, or deteriorate, ICP monitoring with
titrated osmotherapy may be an option with
decompression the next stage. Particular problems are posed by the patient with progressive
swelling, especially posterior shift of the third
ventricle, but preserved capacity to obey commands. For these patients, prophylactic decompressive bifrontal craniectomy is usually
preferable to the risk of sudden death or permanent disability that can result from rapid
herniation.

Bifrontal contusions are present in about 20% of cases of moderate TBI with cerebral contusion and are due to acceleration/deceleration forces pushing the inferior frontal lobes and temporal pole against the irregular skull base. Hemor- rhagic swelling of these contusions may disrupt the median forebrain bundle, gyrus rectus, and anterior hypothalamic nuclei. These structures are involved in behavior control and are associ- ated with changes in personality, volition, motiva- tion, judgment, and social interactions. When these contusions swell, the brain is displaced pos- teriorly (descending transalar herniation) and abrupt deterioration because of descent of the brain stem into the posterior fossa with stretching and deformity of the small perforating blood ves- sels of the basilar artery with risk of death due to respiratory arrest, sudden coma, and autonomic changes. These lesions are also very often associ- ated with disturbances of sodium and water metabolism (e.g. diabetes insipidus or SIADH). Aggressive surgical resection of these contusions may worsen the late neurological deficit and neu- ropsychological consequences. Surgical decom- pression requires bifrontal decompressive craniotomy and cutting of the falx and sagittal sinus on the frontal cranial fossa. This is a major procedure, caries hemorrhage risk, and requires delayed cranioplasty 2-3 months later. The risk of this surgical procedure must, therefore, be bal- anced against risk of death due to herniation. The timing of deterioration is variable, but brain edema will usually peak around the 5th to the 10th day, with resolution of the swelling after this time hence observation for up to 2 weeks with serial CT scanning every 2-3 days is necessary to exclude progression of the lesions. When the patients are unable to obey commands, are very restless, or deteriorate, ICP monitoring with titrated osmotherapy may be an option with decompression the next stage. Particular prob- lems are posed by the patient with progressive swelling, especially posterior shift of the third ventricle, but preserved capacity to obey com- mands. For these patients, prophylactic de- compressive bifrontal craniectomy is usually preferable to the risk of sudden death or perma- nent disability that can result from rapid herniation.

22
Q

Which one of the following AP diameters of a decompressive hemicraniectomy flap is the minimum size thought to prevent local com- plications relating to brain herniation?
a. 10cm
b. 12cm
c. 14cm
d. 16cm
e. 18cm

A

b. 12cm

Unilateral DC (also termed hemicraniectomy) is
usually performed in cases with predominantly
unilateral hemispheric edema—a feature that is
evident on brain imaging as a midline shift to
the contralateral side. Bifrontal DC, which
extends from the floor of the anterior cranial fossa
anteriorly to the coronal suture posteriorly and to
the pterion laterally, is usually performed in
patients with diffuse brain edema. Removal of
the inferior part of the temporal bone to the floor
of the middle cranial fossa is an important maneuver for both types of DC, especially in the presence of temporal pole lesions or edema causing
brainstem compression. It is now well recognized
that, during DC, the dura mater has to be widely
opened as bony decompression alone cannot
sufficiently accommodate severe brain swelling.
Leaving the dura open while covering the brain
with a sheet of hemostatic material (such as
Surgicel®, Ethicon Inc., Somerville, NJ) is our preferred option as it allows for faster closure with a
low chance of complications. If a duraplasty is performed, it should be wide enough to accommodate
further brain expansion. A decompressive hemicraniectomy diameter of 12 cm has been postulated to
represent the minimum size for effective decompression, as the incidence of hemicraniectomyassociated lesions increases sharply with defects
of smaller size. Cerebral herniation, shear stress
along a bony ridge due to steep pressure gradients
as well as compression of cortical veins and aggravated swelling are examples of the mechanisms
thought to contribute to new postoperative contusions, hemorrhage and ischemia. As a consequence,
a minimum diameter exceeding 12 cm is commonly pursued, with adequate decompression to
the floor of the middle cranial fossa being crucial
to prohibit uncal herniation. In a recent study
where all patients had ICP control and all decompressions were >12 cm in AP diameter, no significant difference in outcome or complication rate
was seen in those <18 cm versus those >18 cm
in AP diameter, craniectomies of 12-15 or 18 cm,
though commonly perceived to be somewhat faster
and less invasive and traumatic, were not associated
with a more favorable procedural risk profile, i.e shorter operating time, a lower incidence of structural laceration or transfusion requirements.
Equally, an additional benefit for an exposure that
exceeds 15 or 18 cm could not be found. A small
proportion of craniectomies extended over the
sagittal midline and did not result in a statistically
significant increase in structural laceration or
development of hygroma, but were associated
with significantly more contusions/hemorrhages,
higher rate of meningitis and shunt-dependency,
but also with better GOS outcome

Unilateral DC (also termed hemicraniectomy) is usually performed in cases with predominantly unilateral hemispheric edema—a feature that is evident on brain imaging as a midline shift to the contralateral side. Bifrontal DC, which extends from the floor of the anterior cranial fossa anteriorly to the coronal suture posteriorly and to the pterion laterally, is usually performed in patients with diffuse brain edema. Removal of the inferior part of the temporal bone to the floor of the middle cranial fossa is an important maneu- ver for both types of DC, especially in the pres- ence of temporal pole lesions or edema causing brainstem compression. It is now well recognized that, during DC, the dura mater has to be widely opened as bony decompression alone cannot sufficiently accommodate severe brain swelling. Leaving the dura open while covering the brain with a sheet of hemostatic material (such as Surgicel®, Ethicon Inc., Somerville, NJ) is our pre- ferred option as it allows for faster closure with a low chance of complications. If a duraplasty is per- formed, it should be wide enough to accommodate further brain expansion. A decompressive hemicra- niectomy diameter of 12 cm has been postulated to represent the minimum size for effective decom- pression, as the incidence of hemicraniectomy- associated lesions increases sharply with defects of smaller size. Cerebral herniation, shear stress along a bony ridge due to steep pressure gradients as well as compression of cortical veins and aggra- vated swelling are examples of the mechanisms thought to contribute to new postoperative contu- sions, hemorrhage and ischemia. As a consequence, a minimum diameter exceeding 12 cm is com- monly pursued, with adequate decompression to the floor of the middle cranial fossa being crucial to prohibit uncal herniation. In a recent study where all patients had ICP control and all decom- pressions were >12 cm in AP diameter, no signif- icant difference in outcome or complication rate was seen in those <18 cm versus those >18 cm in AP diameter, craniectomies of 12-15 or 18 cm, though commonly perceived to be somewhat faster and less invasive and traumatic, were not associated with a more favorable procedural risk profile, i.e.
shorter operating time, a lower incidence of struc- tural laceration or transfusion requirements. Equally, an additional benefit for an exposure that exceeds 15 or 18 cm could not be found. A small proportion of craniectomies extended over the sagittal midline and did not result in a statistically significant increase in structural laceration or development of hygroma, but were associated with significantly more contusions/hemorrhages, higher rate of meningitis and shunt-dependency, but also with better GOS outcome.

23
Q

Which one of the following statements regarding decompressive craniectomies is LEAST accurate?
a. DECRA trial has shown the utility of early decompressive craniectomy in neuroprotection
b. DECRA trial utilized decompressive craniectomy at tier 2 management of ICP
c. Primary decompressive craniectomy is usually performed during evacuation of an acute subdural hematoma due to con-
cerns regarding brain swelling
d. Secondary decompressive craniectomy is usually undertaken as a last-tier therapy when a patient has intractable intracranial hypertension
e. Level I evidence of the effectiveness of decompressive craniectomy for refrac- tory intracranial hypertension is still lacking

A

a. DECRA trial has shown the utility of early decompressive craniectomy in neuroprotection

In the modern era of TBI management, DC can
be grouped into two major categories: primary or
secondary. Primary decompressive craniectomy
is usually performed during evacuation of an
acute subdural hematoma (ASDH), either
because the brain is swollen beyond the confines
of the skull or because the patient is thought to be
at high risk of worsening of brain swelling within
the ensuing few days. Secondary decompressive
craniectomy is usually undertaken as a last-tier
(life-saving) therapy when a patient has intracranial hypertension that is sustained at 20-
35 mmHg and refractory to medical management. However, secondary DC can also be
undertaken earlier (that is, before the stages of
last-tier therapy) and in individuals with lesssustained periods of intracranial hypertension.
In such cases, secondary DC can be regarded as
a neuroprotective measure. This was examined
in the DECRA study where 155 adults aged
<60 with diffuse TBI, <72 h post injury and
moderate intracranial hypertension (ICP
exceeded 20 mmHg for more than 15 min within
a 1-h period, and if they did not respond to optimized first-tier interventions) were randomly
assigned to receive either standard medical management alone or medical management plus
bifrontal DC. At 6-month follow-up, the investigators observed a higher rate of unfavorable outcomes in the DC group than in the control group
(70% vs. 51%). However, 27% of patients in the
surgical arm had bilaterally unreactive pupils
compared with only 12% in the control arm. As
pupil reactivity is known to be a major prognostic indicator of outcome following TBI, the investigators performed a post-hoc adjustment for pupil
reactivity at baseline, which revealed that the
between-group difference in terms of unfavorable
outcome was not significant. In contrast to the
DECRA study of early DC for intracranial hypertension, the RESCUEicp trial is examining the
effectiveness of DC as a last-tier therapy for
patients with refractory intracranial hypertension. The RESCUEicp study differs from
DECRA in a number of features: sample size
(400 patients in RESCUEicp vs. 155 patients in
DECRA); surgical technique (bifrontal DC or
hemicraniectomy versus bifrontal DC alone);
threshold for ICP (25 mmHg vs. 20 mmHg); duration of refractory intracranial hypertension (at least
1 h vs. 15 min); timing of randomization (any time
when inclusion criteria are met versus within 72 h
post-injury); and follow-up period (2 years vs.
6 months).

In the modern era of TBI management, DC can be grouped into two major categories: primary or secondary. Primary decompressive craniectomy is usually performed during evacuation of an acute subdural hematoma (ASDH), either because the brain is swollen beyond the confines of the skull or because the patient is thought to be at high risk of worsening of brain swelling within the ensuing few days. Secondary decompressive craniectomy is usually undertaken as a last-tier (life-saving) therapy when a patient has intracra- nial hypertension that is sustained at 20- 35 mmHg and refractory to medical manage- ment. However, secondary DC can also be undertaken earlier (that is, before the stages of last-tier therapy) and in individuals with less- sustained periods of intracranial hypertension. In such cases, secondary DC can be regarded as a neuroprotective measure. This was examined in the DECRA study where 155 adults aged <60 with diffuse TBI, <72 h post injury and moderate intracranial hypertension (ICP exceeded 20 mmHg for more than 15 min within a 1-h period, and if they did not respond to opti- mized first-tier interventions) were randomly assigned to receive either standard medical man- agement alone or medical management plus bifrontal DC. At 6-month follow-up, the investi- gators observed a higher rate of unfavorable out- comes in the DC group than in the control group (70% vs. 51%). However, 27% of patients in the surgical arm had bilaterally unreactive pupils compared with only 12% in the control arm. As pupil reactivity is known to be a major prognostic indicator of outcome following TBI, the investi- gators performed a post-hoc adjustment for pupil reactivity at baseline, which revealed that the between-group difference in terms of unfavorable outcome was not significant. In contrast to the DECRA study of early DC for intracranial hyper- tension, the RESCUEicp trial is examining the effectiveness of DC as a last-tier therapy for patients with refractory intracranial hyperten- sion. The RESCUEicp study differs from DECRA in a number of features: sample size (400 patients in RESCUEicp vs. 155 patients in DECRA); surgical technique (bifrontal DC or hemicraniectomy versus bifrontal DC alone); threshold for ICP (25 mmHg vs. 20 mmHg); dura- tion of refractory intracranial hypertension (at least 1 h vs. 15 min); timing of randomization (any time when inclusion criteria are met versus within 72 h post injury); and follow up period 2 years vs 6 months)